Pls Help! Will Give Brainliest for the right answer!

Pls Help! Will Give Brainliest For The Right Answer!

Answers

Answer 1

a. The survey method is unlikely to result in a representative sample

b. Interview people on all days of the week.

What is a Representative Sample?

A representative sample is a sub-section of a larger population and has similar characteristics of the main group population.

a. The population of the study is the entire consumers that shop at the supermarkets who shop all days of the weeks. Therefore, asking only shoppers who shopped on Monday will most likely give you a bias response as most customers may select Wednesday. The sample doesn't represent the entire population.

b. A better survey method is picking samples on all days of the week and ask them.

Learn more about representative sample on:

https://brainly.com/question/25142707

#SPJ1


Related Questions

A couple plans to have 4 children. The gender of each child is equally likely. Design a simulation involving 55 trials that you can use to model the genders of the children. Write your answer as number

Answers

Answer:

4x55x2

Step-by-step explanation:

4 kids 55trys 50 50 odds

Need help to understand how to do this.

Answers

Answer: -1(x+2)^2+10

Step-by-step explanation:


1. The coefficient of -x^2 is just the number in front, which in this case would be a = -1 So now the function is -1(x^2+4x)+6 since negative one has been factored out of the first two terms.

2. Half of the coefficient of x would be 2 since x has a coefficient of 4, and half pf 4 is 2. Squared, it is 4. That will be added inside the bracket. Subtracted on the outside is the same equation but multiplied by a, which we found out was -1 in the beginning. So, the function becomes -1(x^2+4x+4)+6-(-4).

3. Factor the inside equation to find that it’ll become (x+2)^2, and simplify the outside to get 10. The function will now be;


f(x)=-1(x+2)^2+10


Hope that made sense and make sure to check just in case I did the math wrong :]

Write an equation for the line parallel to the given line that contains C.
C (1,8); 5/7x + 7

Answers

-------------------------------------------------------------------------------------------------------------

Answer:  [tex]\textsf{y = 5/7x + 51/7}[/tex]

-------------------------------------------------------------------------------------------------------------

Given: [tex]\textsf{Goes through (1, 8) and is parallel to y = 5/7x + 7}[/tex]

Find:  [tex]\textsf{Write an equation that follows that criteria}[/tex]

Solution: We know that our equation is going to parallel to the line that was given therefore the slope would stay the same at 5/7.  We also have a point so we can plug in the values into the point-slope form, distribute, and solve for y.

Plug in the values

[tex]\textsf{y - y}_1\textsf{ = m(x - x}_1\textsf{)}[/tex][tex]\textsf{y - 8 = 5/7(x - 1)}[/tex]

Distribute

[tex]\textsf{y - 8 = (5/7 * x) + (5/7 * (-1))}[/tex][tex]\textsf{y - 8 = 5/7x - 5/7}[/tex]

Add 8 to both sides

[tex]\textsf{y - 8 + 8 = 5/7x - 5/7 + 8}[/tex][tex]\textsf{y = 5/7x - 5/7 + 8}[/tex][tex]\textsf{y = 5/7x + 51/7}[/tex]

Therefore, the final equation that follows the description that was provided in the problem statement is y = 5/7x + 51/7.

Select the correct answer. Simplify the polynomial expression. 3x(-2x+7) -5(x-1)(4x-3)

Answers

Answer:

The simplified form is -26x^2 + 56x -15

Step-by-step explanation:

We need to solve the expression:

3x(-2x+7)-5(x-1)(4x-3)

Multiplying the terms outside the bracket with the terms inside the bracket.

=-6x^2+21x-5(x(4x-3) -1(4x-3))

= -6x^2+21x-5(4x^2-3x-4x+3)

= -6x^2+21x-5(4x^2-7x+3)

Now multiply -5 with the terms inside the bracket

= -6x^2+21x -20x^2 +35x -15

Now, Combining the like terms:

= -6x^2 -20x^2 +21x+35x -15

Adding the like terms

= -26x^2 + 56x -15

So, the simplified form is -26x^2 + 56x -15

-26^2+56x-15 yea that should be the answer

helppppp will give brainliest

Determine whether the relationship is an inverse variation or not. Explain.

a The product xy is not constant, so the relationship is an inverse variation.

b The product xy is constant, so the relationship is not an inverse variation.

c The product xy is constant, so the relationship is an inverse variation.

d The product xy is not constant, so the relationship is not an inverse variation

Answers

Answer:

The product xy is constant, so the relationship is an inverse variation.

Step-by-step explanation:

Inverse variation is the mathematical relationship between two variables which can be expressed by an equation in which the product of two variables is equal to a constant.

inverse variation is the relationships between variables that are represented in the form of y = k/x, where x and y are two variables and k is the constant value. It states if the value of one quantity increases, then the value of the other quantity decreases.

Here, the product xy is constant and gives the product as 960.

The relationship is inverse relationship.

Learn more about inverse  variation - https://brainly.com/question/2798700

#SPJ10

Line / has a slope of. The line through which of the following pair of points is
perpendicular to /?
A. (4, 6), (2, 2) B. (8,8), (2, 4)
C. (8,2), (2,4)
D. (6,2), (2, 4)

Answers

The slope of a line.

Let [tex]A(x_A,\ y_A),\ B(x_B,\ y_B)[/tex]. Then a slope of the line AB represent the formula:

[tex]m=\dfrac{y_B-y_A}{x_B-x_A}[/tex]

Substitute the coordinate od the points to the formula of a slope.

[tex]A.\\(4,\ 6),\ (2,\ 2)\\\\m=\dfrac{2-6}{2-4}=\dfrac{-4}{-2}\\\\\huge\boxed{m=2}[/tex]

[tex]B.\\(8,\ 8),\ (2,\ 4)\\\\m=\dfrac{4-8}{2-8}=\dfrac{-4}{-6}\\\\\huge\boxed{m=\dfrac{2}{3}}[/tex]

[tex]C.\\(8,\ 2),\ (2,\ 4)\\\\m=\dfrac{4-2}{2-8}=\dfrac{2}{-6}\\\\\huge\boxed{m=-\dfrac{1}{3}}[/tex]

[tex]D.\\(6,\ 2),\ (2,\ 4)\\\\m=\dfrac{4-2}{2-6}=\dfrac{2}{-4}\\\\\huge\boxed{m=-\dfrac{1}{2}}[/tex]

can someone please help mee (20 points and i will give brainliest!!!)

Answers

Answer:

Step-by-step explanation:

Is (−1, −1) a solution to the following system of inequalities? Show work to verify algebraically. Then, provide an explanation below.

Answers

Yes this is a solution.

A way you can figure this out is by plugging in (-1, -1) to their corresponding values in both equations. The first -1 would replace all x values. The second -1 would replace all Y values.

After simplifying the first equation gives
-1 >= -4 + 3
-1 >= -1 TRUE

The second equations gives

-1 - (-2) >= 1
1 >= 1 TRUE

Can someone take a look at this image and answer please!

Answers

The answers to the following question is

1) KL = 10

2) TE= 15 cm

3) angle PUT= 13

4) angle SQP = 21

What is similarity in triangle?

Two triangles are similar if they have the same ratio of corresponding sides and equal pair of corresponding angles. If two or more figures have the same shape, but their sizes are different, then such objects are called similar figures.

1) Using similarity property in given triangle

SR/ KI= SJ / JK

AS, JK= 2 SJ

SR/ KI= SJ / 2 SJ

5/ KI =  1/2

KL= 10

2) As, the diagonal of parallelogram bisect equally each other then

VE = TE

AS, VE = 15

So, TE= 15 cm

3) As, PU is the bisector of angle SUT

angle PUT= 1/2 (angle SUT)

PUT = 1/2 (26)

angle PUT= 13

4) As PQ is the bisector SQR

angle PQR= angle SQP = 21

Learn more about similarity of triangles

https://brainly.com/question/25882965

#SPJ1

assume that y varies inversely with x. If y= 1.6 when x= 0.5, find x when y= 3.2

Answers

Answer:

x = 0.25

Step-by-step explanation:

If y varies inversely with x, then:

[tex]y \propto \dfrac{1}{x} \implies y=\dfrac{k}{x} \quad \textsf{(for some constant k)}[/tex]

Given:

y = 1.6 when x = 0.5

Substitute the given values into the found equation and solve for k:

[tex]\implies 1.6=\dfrac{k}{0.5}[/tex]

[tex]\implies k=1.6(0.5)[/tex]

[tex]\implies k=0.8[/tex]

Therefore:

[tex]y=\dfrac{0.8}{x}[/tex]

To find the value of x when y = 3.2, substitute y = 3.2 into the found equation and solve for x:

[tex]\implies 3.2=\dfrac{0.8}{x}[/tex]

[tex]\implies x=\dfrac{0.8}{3.2}[/tex]

[tex]\implies x=0.25[/tex]

PLEASE HELP!!! I will give brainliest pleasee help. each question has to be solved, there are no options for answers.

Answers

The intersecting secant theorem states the relationship between the two intersecting secants of the same circle. The given problems can be solved using the intersecting secant theorem.

What is Intersecting Secant Theorem?

When two line secants of a circle intersect each other outside the circle, the circle divides the secants into two segments such that the product of the outside segment and the length of the secant are equal to the product of the outside segment other secant and its length.

a(a+b)=c(c+d)

1.)

6(x+6) = 5(5+x+3)

6x + 36 = 25 + 5x + 15

x = 4

2.)

4(2x+4)=5(5+x)

8x + 16 = 25 + 5x

3x = 9

x = 3

3.)

8x(6x+8x) = 7(9+7)

8x(14x) = 112

112x² = 112

x = 1

4.)

(x+3)² = 16(x-3)

x² + 9 + 6x = 16x - 48

x² - 10x - 57 = 0

x = 14.0554

Learn more about Secant:

https://brainly.com/question/10128640

#SPJ1

Pls help ill give brainiest

Answers

The last two need to be flipped

I am lazy to type so here look at the picture and there is your answer

On Monday, a packaging company put together 450x packages of 45 cashews. On Tuesday, the company put together 175 less packages with 15x more cashews in each bag. Which of the following equations could be used to determine how many cashews were packaged on Tuesday?

Answers

Answer:

275x packages of 720 cashews

Step-by-step explanation:

450-175=275

(a+1)b

a=15 and b=45

(15 + 1) x 45 = 720

Which are true about the pyramid? Check all that apply.
The base of the pyramid is a square with four sides, each measuring 756 feet.
The slant height of the pyramid is 612 feet.
The slant height of the pyramid is 756 feet.
All four triangular faces are congruent.
All four triangular faces are not congruent.
The pyramid has four lateral faces.

Answers

The statements that are true about the pyramid are

The base of the pyramid is a square with four sides, each measuring 756 feet.The slant height of the pyramid is 612 feet.All four triangular faces are congruent.The pyramid has four lateral faces.

Properties of a Square pyramid

From the question, we are to determine the statements that are true about the pyramid

The diagram shows a square pyramid,

The length of a side of its base is 756 ft

and

The height / altitude of a triangular face is 612 ft

Since the pyramid is a square pyramid,

Then, the base must be a square with each side measuring 756 feet

∴ The first statement is true

Since the height of a triangular face is 612 ft,

Then,

∴ The slant height of the pyramid is 612 feet

NOTE: Slant height of a pyramid is the altitude of the triangle comprising a lateral face

Thus, the second statement is true

Since the triangular faces each have a base of 756ft and a height of 612 ft,

Then,

All four triangular faces are congruent

∴ The fourth statement is true

NOTE: Lateral sides of a solid are the faces on the sides of the shape

The pyramid has four lateral faces which are each a triangle

∴ The last statement is true

Hence, the statements that are true about the pyramid are

The base of the pyramid is a square with four sides, each measuring 756 feet.The slant height of the pyramid is 612 feet.All four triangular faces are congruent.The pyramid has four lateral faces.

Learn more on Pyramid here: https://brainly.com/question/10042135

#SPJ1

I need a written answer for all three questions please.

Answers

The values of the trigonometry ratios are:

cos α = - 5/13 and cot α = 5/12cot α = -5/12 and sec α = 13/5

How to solve the trigonometry ratios?

1: sin α = -12/13 and tan α > 0, find cos α and cot α

Because tan α > 0, then it means that cos α and sin α are negative

So, we have:

sin²α + cos²α = 1

Substitute sin α = -12/13

(-12/13)² + cos²α = 1

This gives

cos²α = 1 - (-12/13)²

Evaluate the squares

cos²α = 1 - 144/169

Evaluate the difference

cos²α = 25/169

Take the square root of both sides

cos α = - 5/13

The cotangent ratio is represented as:

cot α = cos α/sin α

This gives

cot α = (-5/13)/(-12/13)

Evaluate

cot α = 5/12

Hence, cos α = - 5/13 and cot α = 5/12

2: tan α = -12/5 for α in quadrant IV, find sec α and cot α

Because α is in quadrant IV, then it means that sec α is positive

cot α = 1/tan α

This gives

cot α = 1/(-12/5)

Evaluate

cot α = -5/12

Also, we have:

sec²α = 1 + tan²α

Substitute tan α = -12/5

sec²α = 1 + (-12/5)²

Evaluate the squares

sec²α = 1 + 144/25

Evaluate the sum

sec²α = 169/25

Take the square root of both sides

sec α = 13/5

Hence, cot α = -5/12 and sec α = 13/5

Read more about trigonometry ratios at:

https://brainly.com/question/11967894

#SPJ1

The set of life spans of an appliance is normally distributed with a mean mu = 48 months and a standard deviation sigma = 8 months. what is the z-score of an appliance that stopped working at 64 months?

Answers

The z-score of an appliance that stopped working at 64 months is 2.

What is mean?

The mean of observations is equal to the ratio of sum of all the observations and the number of observations.

Given, the set of life spans of an appliance is normally distributed with a mean mu = 48 months and a standard deviation sigma = 8 months

The z-score is then calculated as

z = x - μ /σ

For an appliance that stopped working at 64 months, we have

x = 64

On substituting the values, we get

z = 64-48/8

z = 2

Hence, the z-score of an appliance that stopped working at 64 months is 2

Learn more about mean.

https://brainly.com/question/11822836

#SPJ1

Answer:

D on edge

Step-by-step explanation:

2

Given the matrices A and B shown below, find -A + 1/3B

Answers

Step-by-step explanation:

1. first multiply -1 times all elements of matrix A

2. then multiply 1/3 by all elements of matrix B

3. then add each corresponding entries to get the result.

from step 1. matrix A will be

-4 -2. -1. -3

-2. 0. 1. -3

step 2. matrix B will be

3. -1. -2. -4

3. -10. 10. -1

add each corresponding elements to get

-1. -3. -3. -7

1. -10 11. -4

3 x 2/5 converted into a mixed number

Answers

Answer:

[tex]1 \frac{1}{5} [/tex]

Step-by-step explanation:

[tex]3 \times \frac{2}{5} \\ \frac{3}{1} \times \frac{2}{5} = \frac{6}{5} \\ \frac{6}{5} = 1 \frac{1}{5} [/tex]

Describe the
translation.
y = (x - 5)² +5 →
y = (x −0)² +0
A.T<5,-5>
OB.T<-5,-5>
OC.T<-5,5>
OD. T<5,5>

Answers

The translation of the parabola from y = (x – 5)² + 5 to y = (x − 0)² + 0 will be (-5, -5). Then the correct option is B.

What is the parabola?

It's the locus of a moving point that keeps the same distance between a stationary point and a specified line. The focus is a non-movable point, while the directrix is a non-movable line.

The equation of a quadratic function, of vertex (h, k), is given by:

y = a(x – h)² + k

where a is the leading coefficient.

The translation of the parabola is given below.

y = (x – 5)² + 5 → y = (x − 0)² + 0

Then the translation will be (-5, -5).

Then the correct option is B.

More about the parabola link is given below.

https://brainly.com/question/8495504

#SPJ1


Determine if the sequence 4, 10, 19, 31,... is arithmetic. If it is, determine the first term a and the common difference d

Answers

Answer:

It is not arithmetic, so therefore there is no common difference

Step-by-step explanation:

An arithmetic sequence is a sequence where each term increases by adding/subtracting some constant k. There is no constant term in the above pattern.

Answer:

  not an arithmetic sequence

Step-by-step explanation:

An arithmetic sequence is a sequence of numbers that have a common difference. That is, the difference between any term and the previous term is constant for the sequence. Other kinds of sequences have other relationships between the differences.

Differences

The "first" differences of this sequence are ...

10-4 = 619-10 = 931 -19 = 12

The first differences are not constant. However, we notice the "second" differences are constant. These are the differences of successive first differences.

9 -6 = 312 -9 = 3 . . . . . . constant 2nd differences

Sequence type

The first differences are not constant, so this sequence is not an arithmetic sequence.

For polynomial sequences, the level of constant difference tell you the degree of the polynomial describing the sequence. This sequence has constant 2nd-level differences, so can be described by a 2nd degree (quadratic) polynomial:

  f(n) = 1.5n² +1.5n +1 . . . . . a quadratic sequence

__

Additional comment

Sequences that are exponential have differences that have a common ratio. That ratio is the same at every level. It is the base of the exponential function.

Which function represents exponential growth?
O f(x) = 3x
O f(x) = x³
Of(x) = x + 3
O f(x) = 3x

Answers

Answer:

b step by step explanation

The function that represents exponential growth is:

f(x) = x³

What is Exponential Growth?

An exponential function's curve is created by a pattern of data called exponential growth, which exhibits higher increases over time.

y = a(1+r)ˣ,

where a is the initial population and r is the rate in decimals and x is the time period.

Exponential growth is a phenomenon that occurs when the rate of change of a quantity is proportional to its current value. In other words, as the value of the quantity increases, the rate at which it increases also increases. This can be represented mathematically using an exponential function of the form f(x) = abˣ, where a is the initial value, b is the growth factor, and x is the time or number of periods.

The function that represents exponential growth is:

f(x) = x³

To learn more about the exponential growth;

https://brainly.com/question/12490064

#SPJ7

A mapping diagram showing a relation, using arrows, between input and output for the following ordered pairs: (negative 3, negative 9), (2, negative 6), (negative 5, 4), (1, 2), (6, 0).
What is the domain of the function shown in the mapping?

Answers

Answer:

{-3, 2, -5, 1, 6}

Step-by-step explanation:

The domain is the set of input values.


Parallelogram EASY is drawn with diagonal ES. The measure of Angle AES is 40 degrees and the measure of Angle Y is 110
degrees.
Find the measure of Angle A, Angle YEA, Angle ESA, and Angle ESY.

Answers

Answer:

Step-by-step explanation:

Givens

Parallelogram EASY with diagonal ES

<AES = 40

<Y = 110

Solution

<AES = 40               GIVEN

<ESY = 40                     Z THEOREM OF A PRALLELOGRAM

<A = <Y                         PROPERTY OF A PARALLELOGRAM

<A = <110                      <A = 110 BECAUSE <Y = 110

<YES = 180 - <Y           -<ESY EVERY TRIANGLE HAS 180o

<YES = 180 - 110 - 40    SIMPLIFY

<YES = 30

<YEA = <YES + <AES    WE KNOW BOTH ANGLES ON THE RIGHT.

<YEA = 30 + 40             COMBINE

<YEA = 70

You could do <YEA by noting that consecutive angles of a Parallelogram equal 180

Suppose the u.s. government put a 'special 20 percent luxury tax' on the retail price of expensive and fancy yachts in order to collect more taxes from boat owners. assume the price elasticity for these yachts is elastic at 2.50. conclusion: we can probably expect that yacht sales will go down and the government will not collect lots of new tax revenues.

Answers

True. We can probably expect that yacht sales will go down and the government will not collect lots of new tax revenues.

It is given that the US government has put a 20% luxury tax and the price elasticity for these yachts is elastic at 2.50.

Due to the new tax on luxury goods, it can be estimated that the sales of yachts go down as the maximum retail price of the yachts will become more expensive. This can mean that fewer people than before will be able to afford the yachts. Due to fewer sales, the government will not have enough tax revenues.

To learn more about price elasticity visit: https://brainly.com/question/4610585

#SPJ4

Q2 - Using fractions
Jarvis works in a garage for $7 an hour.
If he works on Saturday he is paid time and a quarter.
If he works on Sunday he is paid time and three quarters.
Last weekend Jarvis worked for four hours on Saturday and four hours on Sunday.
How much was Jarvis paid last weekend altogether?

Answers

Total Earned Saturday: $45

Total Earned Sunday: $63

Total: $108

Time and a quarter means that you divide his normal rate by 4 and add that to his normal rate.

9/4 = 2.25     2.25 + 9 = 11.25

Jarvis makes $11.25 an hour on Saturdays.

Since he worked 4 hours, you multiply,

11.25 x 4 = 45.

What is the normal rate?

Time and three quarters mean that you multiply his normal rate by 3/4 and add that to his normal rate

9x0.75 = 6.75     6.75 + 9 = 15.75

Jarvis makes $15.75 an hour on Sundays. Since he worked 4 hours, you multiply 15.75 x 4 = 63.

To get the total, you add his earnings on Saturday and earnings on Sunday

45 + 63 = 108.

To learn more about the using fractions visit:

https://brainly.com/question/78672

#SPJ1

Peter uses the equation y = StartFraction 13 over 4 EndFraction x to model the number of miles that he has walked in x hours. Which statement is true about the proportional relationship that is modeled by Peter’s equation?

Answers

Regarding the proportional connection that Peter's equation simulates, claim C is accurate. Peter moves along at a 13/4 mph walking pace.

What is the equation?

An equation is a mathematical statement that uses an equal sign to join two algebraic expressions having the same value.

The complete question is

"Peter uses the equation Y=13/4x to model the number of miles that he has walked in x hours. Which statement is true about the proportional relationship that is modeled by the peat there's the equation?

A:Peter walks a rate of 4/13 miles per hour.

B: Peter walks at a  rate of 4 miles per hour.

C: Peter walks at a rate of 13/4 miles per hour.

D: Peter walks at a rate of 13 miles per hour."

Provided equation

Y=13/4x

where y is the distance traveled and x denotes the amount of time.

According to the equation, Peter moves along at a speed of 13/4 miles per hour.

As a result, the proportional connection represented by Peter's equation is valid as stated in assertion C.

To learn more, about equations, refer;

https://brainly.com/question/10413253

#SPJ1

Hi can someone assist me with this question and help me solve it?

Answers

Answer:

40°

Step-by-step explanation:

Given l \\ m,

m∠13 = m∠7 (alternate interior angles)

m∠13+m∠15 = 180° (Sum of angles in a straight line)

[tex]6x+4+(14x-4)=180\\6x+4+14x-4=180\\20x=180\\x=\frac{180}{20} \\=9\\\\[/tex]

Substitute x to find m∠13

m∠13= 6x+4 = 6(9)+4 = 36 + 4 = 40°

5)Express in standard form: (i)4809000 (ii)0.00849​

Answers

Answer:

I hope it helps .in case you don't understand any part ,feel free to ask.

How do you solve this?

Answers

Step-by-step explanation:

to "solve" this I need an equation.

this whole expression must be equal to something.

without that I can only try to simplify the expression.

remember that

a/b / c/d = ad / bc

so, here we have

3a/(((a²/x) - 1)(a/x - 1)) = 3a/(a³/x² - a²/x - a/x + 1) =

= 3a/(a³/x² - a²/x - a/x - a/a) =

= 3a/((a²/x² - a/x - 1/x - 1/a)×a) = 3/(a²/x² - a/x - 1/x - 1/a)

What is the multiplicative rate of change of the function? two-thirds three-fourths four-thirds three-halves

Answers

The multiplicative rate of change is 2/3

How to determine the multiplicative rate of change?

The complete question is in the image

The multiplicative rate of change is then calculated as:

r = y2/y1

This gives

r = 4/6

Simplify

r = 2/3

Hence, the multiplicative rate of change is 2/3

Read more about rate of change at:

https://brainly.com/question/4319809

#SPJ1

Answer:

b.

Step-by-step explanation:

Other Questions
which statement by breast-feeding mother indicates that the nurse's teaching regarding stimulating the let-down help me fill this page with the right answer please (17 lines minimum please) why open source software is very important for the country like Nepal Explain how to determine if Carloss allowance is directly proportional to the number of chores he completes. Match the term in column 1 to the characteristics in column 2.Ice sheetMoraineContour plantingRetreatingOne way to prevent soilerosionShrinking in a glacierRock piles left by glaciersContinent sized In his winning campaign of 1980, _____ claimed that the federal government, in its attempts to improve society, was actually eroding individual freedoms. classify the following into colloids, suspensions and true solution: soda water,chalk powder,milk,blood,ink,starch solution,sulphur powder in water,egg albumin. Based on the audio, which statement is correct about what Alana does to get ready for her birthday? She has her hair done first and does her nails last. She puts on makeup first and does her hair last. She shaves her legs first and does her makeup last. She takes a shower first and dresses in pink last. at the household level, higher saving rates lead to ______ and greater economic security. Which of the following statements about psychological constructs is true?A.Psychological constructs are easy to define and measure.B.Psychological constructs represent concepts that cannot be observed or measured directly.C.Psychological constructs cannot be researched in psychology because they are unobservable.D.Psychological constructs represent measurable overt behavior.Please select the best answer from the choices providedABCD On tax-free weekend, Ben buys school supplies totaling $47.50. He has a sale coupon for 15% off his entire purchase. What will Ben final cost be after the 15% discount? In what year did juneteenth officially become a federal holiday?. please help me solve this LCM Given that 2x 1 4 = 16x + 1 2x 1 find the possible values of Drag each label to the correct location.Identify the character, problem, and solution in Kareem's Problem.Drag each statement to the correct location on the chart. Not all statements will be used.Mrs. McMudge is avery kind personwho speaks sosoftly that no onecan hear her.Jenny informs Kareemthat Mrs. McMudgedoesnt yell to bemean but becauseshe cannot hear well.JennyKareem doesnt likedelivering packagesto Mrs. McMudgebecause she yells,and he thinks sheis mean.Kareem-Main Character-Problem-Solution HELP MEEEEEEEEEEEEEEEEEEEEEEEEEEEEEEEEEEEEE write and solve a proportion to complete the statement. Round to the nearest hundredth if necessary.1. 6km ?mi2. 2.5 L ?gal3. 90lb ?kgpls help Input 1,2,3,4,5, n output 2,-1,-4,-7, ?please help me find the linear function! For the statement below, fill in the blank with the answer that makes the statement true.In the House of Representative, votes are based on the statespopulationfinances and wealthpopularitydate it was establish The equation for free fall at the surface of some planet (s in meters, t in seconds) is s=2.86t^2. How long does it take a rock falling from rest to reach a velocity of 25.1 m/s on this planet? Consider the line 4x-8y= -6. What is the slope of a line perpendicular to this line? What is the slope of a line parallel to this line